Hace 6 años | Por maria1988
Publicado hace 6 años por maria1988

Comentarios

D

#65 La mía de #64 no es muy currada... pero diría que es correcta y extensible a Rn
P.S.: Y si no es correcta aun así tiene mérito haber escrito ese tocho con un móvil

D

#67 Extra mérito lol

D

#67, ¿cómo la ampliarías a Rn?

D

#73 Usando el mismo método tengo que para todo Rn puedo coger n-1 puntos tales que una transformación lineal de sus segmentos de longitud irracional me de un número irracional a la potencia de n. Lo tengo en la cabeza y no soy capaz de explicarlo... pero saqué el del plano porque conseguí el de la línea con dos puntos. No debería ser complejo demostrarlo para todo n... creo. Y si no al método informático: todo vector de Rn se puede transformar a un vector de Rn-1

D

#64 , uhm, en realidad lo que has hecho ha sido calcular

BP2+CP2-2AP2

con lo que terminas demostrando que uno de los 3 es el la raíz de un irracional, pero vamos, al final valdría, creo, espera que lo mire bien. Y efectivamente tu prueba sería más sencilla que la mía.

D

#71 perdón eso fue por el móvil. Me comí los cuadrados. Es una transformación lineal. Si r^2 es irracional entonces 2r^2 lo es. Si es así, entonces AP BP o CP, alguno de ellos es irracional. Con lo cual al menos uno de esos 3 segmentos me ha tachado el punto P sea el que sea. Con lo cual esos 3 puntos cubren todo el plano con sus segmentos de longitud irracional.

D

#72, elevar al cuadrado no es una transformación lineal

Pero que al final deduces que AP2, BP2 o CP2 es irracional, y por tanto al hacer raíz cuadrada también y la demostración valdría si no nos hemos colado en nada, claro.

D

#75 Pues como se llame... soy informático no me pidas mucho... somos así medio monguers por naturaleza

D

#72 no hagas que r sea raíz de 2, o se te desmonta el chiringuito

maria1988

#64 No lo he mirado con detenimiento, pero tiene buena pinta. La solución son tres puntos, pero, personalmente, nunca se me ocurrió que poniendo los tres en la misma recta y tan simétricos funcionara.

fantomax

#0
Muy buen problema!

maria1988

#12 También hay infinitos puntos a distancia irracional de uno dado.
La solución no es correcta, el número de pinchazos necesarios es mucho más pequeño.

D

#13 ok pues entonces es dos, y no te la razono por que es una intuición... Y he de reconocer que las mates no son lo mío.
Un saludo.
Y gracias.

maria1988

#14 Yo también pensé 2 cuando me plantearon el problema. En #8 he explicado por qué esta solución no es correcta.

jonsy.gaviota

#126 Me desdigo: había interpretado que los alfileres se pinchaban al azar... sorry

Aplicando el razonamiento de #8 y de #15, si disponemos de tres alfileres, situados a distancia irracional cada uno de los otros dos, tendremos para cada par de alfileres una recta que no queda cubierta de rojo... pero que sí quedaría cubierta por el tercer alfiler. Para ello no tiene que haber ninguna relación racional entre las distancias de los tres alfileres... vamos, que las tres medianas no se corten en un punto común

En ese caso, con tres alfileres bastaría

Wayfarer

#13 Es uno de esos problemas con infinitos de diferentes tamaños...

Yo sigo pensando que se requerirían infinitos pinchazos, pero esperaré a queFantomaxFantomax explique por qué estoy equivocado.

D

#17, si #13 no ha mentido son menos que infinitos. Y si no me he equivocado yo en lo que acabo de pensar efectivamente son menos.

Y lo de infinitos distintos. El conjunto de números irracionales es igual de grande que el conjunto de los números reales, vamos, que el infinito de puntos coloreados con tan solo un pinchazo es un infinito igual de grande que el del puntos del mantel.

Si la pregunta llega a ser pintados con distancia racional sí que saldría directamente infinitos usando lo de tamaños infinitos distintos, pero no es el caso, y sería pasarse poner un problema en el que se use eso

P.d. #0, ya lo tengo si no me he equivocado, pero tengo que decir que he tenido que usar números trascendentes, seguro que la solución que tú tengas es más sencilla. Por cierto, ¿has pensado el mismo problema en Rn?

fantomax

#29 Así, a lo sencillote, trascendentes.

D

#33, es que en un triángulo rectángulo que por ejemplo los catetos sean racionales no implica que lo sea la hipotenusa, pero sí puedes asegurar que la hipotenusa vaya a ser algebraica y si he partido de números trascendentes en tales sitios puedo llegar a deducir que el punto investigado está a distancia trascendente de alguno de mis puntos iniciales y en particular irracional y por tanto estaría coloreado.

¿Tú lo has conseguido sacar ya (y de forma relativamente sencilla) o algo?

fantomax

#35 tengo una cota pero no lo tengo matado.

maria1988

#29 Buah, fijo que has pensado en una solución mucho más compleja que la mía. Creo que la mía es extensible a Rn.

D

#40, la mía creo que también es extensible a Rn. Pues tengo ganas de ver tu solución.

maria1988

#56 Se necesitan tres puntos. En Rn, la solución es n+1. Espero que hayas llegado al mismo número, porque si no fijo que es la mía la que está mal. De todas formas te aviso que mi razonamiento no tiene todo el rigor que debería.

D

#61, pero no te chives, deja a esta gente que siga calentándose la cabeza. Y sí, en este caso coincido contigo (aunque he hecho los razonamientos en 2 post-it, todo riguroso pero podría haberme colado en algo) y coincido también en la que debe ser en Rn, de hecho para n=0 y n=1 claramente es cierto.

D

#61 #62 #59 Propongo poner en el enunciado del SUB el grado de dificultad o al menos los conocimientos mínimos necesarios para resolver. 60 horas de "Análisis Matematico" con aprobado justito hace 30 años, no da para esto (pero entretiene igual)

Es el SUB que me leo con el café mañanero

maria1988

#63 En serio que no hacen falta muchos conocimientos previos para resolverlo, al menos no si tu solución es la patatera que tengo yo. Seguramente la de@zurditorium estará más currada.

D

#63 Bueno... yo no estudié matemáticas, hice informática, el cálculo numérico, álgebra lineal y matemática discreta suficientes como para no cagarme en los desfiles... pero me gustan los retos porque me mantienen el cerebro entretenido, cuanto más difíciles mejor, así no pienso en otras cosas

fantomax

#17 Con respecto a llamarme,maria1988maria1988 es tan matemática como yo. Vamos siendo unos cuantos por aquí.
Con respecto a infinitos más grandes, a la primera que pinchas cubres de rojo una cantidad no numerable de puntos y sólo queda blanca una cantidad numerable.

Wayfarer

#30 No sabía lo de María1988, gracias. Y gracias también por la explicación.

maria1988

#42 Ya, pero yo soy una matemática con poca práctica, que desde que acabé los estudios prácticamente solo he tocado la estadística.
cc. #30

D

#47 Ortega y Gasset, habla de filosofía, pero es aplicable a todos los campos: "La diferencia entre un filósofo de carrera y uno que no lo es, es cuantitativa, no cualitativa"

maria1988

#7 Ay, perdona, no había leído el otro comentario.

La solución no es correcta.

Explicación: Si pinchas dos veces, en la mediatriz del segmento que une los dos puntos donde has clavado el alfiler tienes infinitos puntos que están a una distancia racional de ambos pinchazos.

Es decir, sean A(A1,A2) y B(B1,B2) los puntos en los que has pinchado:
1. Unes los dos puntos en los que has pinchado con una línea imaginaria.
2. Haces la mediatriz del segmento. Todos los puntos de la mediatriz tienen la propiedad de que son equidistantes a A y B. Si la distancia entre A y B es D, los puntos de la mediatriz estarán a distancias [D,infinito) de A y de B.
3. Así, tienes un número infinito de puntos a distancia racional de A y B, que seguirán siendo blancos.

D

2 no son.

Imaginad que pinchamos con 2 puntos a y b. Sea N un número natural más grande que la distancia entre los dos puntos. Tracemos una circunferencia de radio N centrada en cada uno de sus 2 puntos y quedémonos con sus puntos de corte. Cojamos uno de los 2, llamémoslo c. El punto c estará a una distancia racional tanto de a como de b (a distancia N), por lo tanto este punto no estaría teñido de rojo.

Yo tampoco lo he sacado, creo que casi lo tengo, pero a los matemáticos el casi no nos vale

editado:
anda, que en #8 ya habías dicho más o menos lo mismo que yo.

D

#8 Pues... Así a lo loco diré aleph sub 0. Mismo planteamiento, pero aplicando un algoritmo similar al planteado en el hotel de hilbert para moverme por Q.

maria1988

#53 No, el número de puntos necesarios es finito.

D

#57 Bien! eso significa que puedo seguir pensando! Me encanta

maria1988

#89 No. Hay que demostrarlo para 3 y dar un contraejemplo para 2. Mi contraejemplo eran los puntos de la mediatriz, como explico en #8.

maria1988

#87 No es correcto. Ve a #8

D

#45, bueno, realmente es que en medida se usaba "almost everywhere" y en conjuntos "almost very point"

D

Infinito ?

maria1988

#10 Como he dicho, no vale con decir un número, hay que razonar la respuesta.

D

#11 ok... Si el mantel es infinitamente grande , tendrás que marcar un número infinito de veces ?
Yo creo que van por ahí los tiros.

fantomax

#28 En este caso estamos moviéndonos entre los dos cardinales infinitos más conocidos, aleph_sub_cero y el cardinal del continuo, este segundo estrictamente mayor que el anterior. Cuando pincho la primera vez tiño de rojo el cardinal del continuo y dejo sin teñir aleph_sub_cero

D

#54 Espero un resultado simple y sorprendente.
Esto es mejor que una peli de intriga

D

#88, ¿no te habré fastidiado tu solución?

D

#81, circunferencia de radio cualquier número racional (por ejemplo 4) mayor a pi centrada en un punto, y otra circunferencia centrada en el otro punto con otro radio racional mayor que pi distinto al anterior (por ejemplo 5). Donde se corten se queda en blanco.

maria1988

#84 ¡Es verdad!

D

#25 Se pintan "todas" las distancias irracionales. El mantel es infinito, y también los son los círculos rojos del primer pinchazo. De hecho, cada pinchazo, genera una serie infinita de círculos... (y no pienso decir el tipo de infinito ...)

D

#3uff.. no sé si irán por ahí los tiros. Yo dije 2, desde el móvil, porque lo he pensado así: si pincho en la (0,0) tiño todos los irracionales. Pero si ahora pincho el (2^.5, 2^.5)... tiño todos los racionales porque es lo mismo que mover el origen de coordenadas a ese punto, así que teñiría todos los "irracionales" con respecto a un irracional.

fantomax

Seguiré pensando en un parque infantil. Adioses.

D

Tal vez 2?

maria1988

#2 No basta con decir un número, hay que explicar cómo se ha llegado a esa solución. Si me dices por qué crees que es dos, te digo por qué es o no correcto.

D

#6 Lo he dicho más abajo Si clavo en (n,n) me deshago de todos los puntos a una distancia irracional de n (dicho en tu enunciado). Si ahora clavo en (n+x, n+x) dónde x es irracional, cambio las coordenadas del sistema para estar en (x,x) así que en este nuevo origen el punto (n+x, n+x) es el punto (n, n), con lo cual si lo pincho quito todo los irracionales con respecto a este punto... que al ser un irracional lo que estoy pintando es todos los racionales.

fantomax

#23 hay distancias irracionales arbitrariamente grandes, cada vez que pincho se tiñe de rojo una región no acotada del plano

R

#24 El mantel sigue siendo infinitamente grande, con lo cual nunca estará completamente teñido de rojo porque siempre va haber infinitamente mas blanco que rojo por mucho rojo que pongas.

fantomax

#25 No, de hecho en cuanto pinchas en un punto se tiñen casi todos los puntos.

R

#26 Si es infinito no existe el casi todo ni el todo, por lo cual nunca será rojo por completo, de echo siempre será infinitamente mas blanco que rojo.

D

#28 Entonces jamás sería blanca... si no puedes hacer todos los puntos rojos, tampoco puedes hacer todos los puntos blancos.

R

#93 Se trata de ponerlo completamente en rojo.

D

#98 Si pero si ser infinito es la única condición, también sería un impedimento para hacerla blanca. Si alguien la psuo blanca, otra persona la podrá poner roja... ¿Cómo? Ahh amigo... ni puta idea.

El otro dia pusieron un problema que resolví y me puse chulo... pero este me supera.

D

#26, buuuuu, en matemáticas cuando se habla de "casi todos los puntos" siempre era todos los puntos menos una cantidad finita (aunque esta cantidad fuera un trillón). Todos menos una cantidad numerable no es casi todos, pero vamos, te he entendido

fantomax

#38 el conjunto que no se cubre tiene medida cero, así usaba yo "casi todo punto" en teoría de la medida.
PS:¿Seguro que no estamos echando a la gente con tanto tecnicismo?

D

#41, es verdad, en teoría de la medida sí, me he colado

fantomax

#44 ñigoñigoñigo...
Joder, estudié eso antes de la revolución industrial, creo.

D

#41 Nah.. acabo de buscar "números trascendentes" en google....

fantomax

#95 ¿Y se entiende bien?
Hay que saber que los trascendente son el grueso de los números reales, y que sus representantes máximos (e y π ) son imprescindibles.

D

#97 No. jajaja... pero es porque es primera lectura... falta base e investigar, tiempo. Me lleva mucho tiempo estudiar, soy lento de cojones. Lo investigué por curiosidad... pq hasta la fecha no he encontrado un conjunto con más complejidad que los reales (o los hiperreales), los demás son construcciones contando con ellos. Los que he "olido".

Por ejemplo, los complejos son RxR, y los trascendentes son un subconjunto de R.

Pero mola escucharos referencias... te enteras de cosas.

fantomax

#99 No, la complejidad de los reales es baja en realidad, el conjunto realmente complejo y para el que hay proposiciones indecidibles son los naturales (estoy hablando de teorema de Gödel y este nivel de lógica de modelos).

D

#97 ¿Que diferencia hay entre un trascendente y un irracional?

D

#38 Proporción "supuesta" entre N sub 1 y N sub cero???
Si te quitas los primeros, puedes decir tranquilamente que has quitado más del 99% de los puntos...

editado:
NO ES LO QUE YO PIENSO... sigo vuestro juego.

D

Me voy un minuto y hay 39 comentarios. Ole! Me está encantando el problema

D

#49 Yo estoy dandole al F5 (me he rendido despues de apostar por el 5)

D

#50 Deberías tenere cuidado con las rimas. Pon mejor 6 y te evitas problemas

D

#54 6! Por el culo os la hinco, no desesperéis!

D

Entiendo que si pinchas en el punto (0.0) por ejemplo. Todos los puntos a una distancia de "por ejemplo" raiz(2) y sus múltiplos quedan teñidos. Los catetos son la unidad, así que... pocos

Me encantan estos! (y si no acierto, a la tarde sigo)

maria1988

#1 Si no quieres pistas, no sigas leyendo.

[SPOILER ALERT]

Ten en cuenta que el sistema de referencia es el que tú elijas. Está claro que el mantel es R^2 y puedes suponer que el primer punto en el que pinchas es el origen. Evidentemente, si pinchas solo una vez, se tiñe de rojo I^2, pero queda blanco Q^2 (que, si bien es infinitamente más pequeño, es denso en R^2). Así, está claro que con clavar una vez el alfiler no es suficiente.

Dikastis

#5 Jodermaria1988maria1988, cómo estás últimamente... que si los verdes y los azules que si alfileres que tiñen de rojo...

Dedícate a planchar el mantel y calla la boca joder, que no haces más que dar problemas!

P.S.: Es una completa broma... solo que no se qué decir ya que mi ignorancia me impide prácticamente intentar siquiera abordar este tipo de problemas... cómo me gustaría poder dedicarme nada más que a tratar de aprender de todo...

Bss y gracias por tratar de aportar conocimiento al mundo!

D

Dados dos "catetos" naturales consecutivos (n) y (n+1), entre sus cuadrados (n^2) y ((n+1)^2) siempre se puede encontrar un número primo (p). Y resulta que las raíces cuadradas de los números primos son números un tanto particulares. La raíz cuadrada de cualquier número primo es un número irracional.

Sigo buscando...

maria1988

#3 No, no tiene nada que ver con eso. Para resolverlo no hace falta emplear ningún método complicado, es un problema más bien intuitivo.

D

Pues apuesto por Fluffy. Dos puntos infinitamente (o suficientemente) alejados.
Los puntos rojos se incrementan con la distancia del pinchazo hasta que llegan a ser todos.

D

#18 El infinito de los irracionales es mas grande que el de los racionales... Si. Voto por los dos puntos lejanos.

fantomax

#22 ya han demostrado que más de dos.

D

Cinco. Dos puntos dejan una recta. 4 dejan un punto y uno mas lo elimina.
Y me rindo, que soy de letras

D

#36, 2 puntos no dejan una recta, dejan muchos puntos fuera de la mediatriz también.

maria1988

#55 ¿Seguro? Si pincho en un punto y después en otro que está a distancia pi, ¿qué puntos se me quedan blancos fuera de la mediatriz del segmento que los une?

maria1988

#36 No es correcto.

D

#68 Ganas de votarte negativo aumentando exponencialmente lol

squanchy

Con que pinches una vez vale. El ojo humano lo va a ver todo rojo porque no tiene tanta precisión como para apreciar los puntos blancos. Hala, matemáticos, JAQUE MATE.

maria1988

#39 Tienes un número infinito de puntos en blanco. Es más, en cualquier pedacito de mantel, por pequeño que sea, habrá un punto blanco. Es lo que tiene la densidad de Q en R.

D

1.- si la sabana es infinitamente grande, cualquier punto es el centro.

2.- Si pincho en un solo sitio, tengo círculos cuyo radio es Irracional. Con solo pinchar una vez, ya cubro todos las posibilidades Irracionales.

3.- Si quisiera cubrir TODAS las posibles distancias reales, tendría que cubrir las distancias tipo Q.

4.- NOTA IMPORTANTE Un Irracional mas un Irracional no tiene pq ser Irracional!!
Dado un Irracional, su "complemento" ( llamemos así al numero que si se lo sumo me da un numero entero) también lo es.. FUCK!!!

5.- Por mucho que la doblemos, pintar la mitad y pintarla toda es lo mismo. Saco el tema de doblarla como necesidad del problema.

6.- Hmmm... la propiedad 4 la apunte a lo loco (pq me jodió el primer intento) y mira por donde... molaaaa.

Dado un Irracional, puedo sumarle el nuevo concepto de complemento... que va a consistir en lo siguiente:

Dadas las cifras decimales de un Irracional, el conjunto de los irracionales que sumados o restados a él que me dan un Racional. Ejemplo:

3,1416
0,8583... como el de arriba no sigue patrón, el de abajo tampoco. Y lo podemos hacer para que cuadre con muchos Racionales....
Obtenemos 3,999999 periodico, que es el 4. Racional.

Si ahora pincho en alguna parte de los círculos que obtuve en el primer pinchazo... Puedo cubrir puntos que estaban a una distancia racional del primer centro.


Y aquí me rindo con mis pistas.

Espera.. no me rindo... ¿Dado un racional puedo encontrar un Irracional del que sea su complemento? SIP!! Es una ecuacion sencilla...

Cota máxima de pinchazos necesarios: El cardinal de Q + 1

maria1988

#74 [SPOILER] El resultado es un número finito, no necetias pinchar tantas veces.

D

#79 Me quedé flipando al descubrir la propiedad del complemento... y ya me rindo.. ... por ahora jajajaja.

D

Bueno, no se si es tan sencilla...

maria1988

#76 A decir verdad, soy una entusiasta de los problemas con enunciados sencillos y soluciones difíciles. Son los que más me gustan, sobre todo si no necesitas conocimientos avanzados de matemáticas para resolverlos.

D

#82 enunciado sencilo.. hmmm, solución dificil: el probelma por antonomasia (no se que significa, pero siempre mola ponerlo al final): "Arreglar el mundo."

R

Muy fácil: infinitas.

fantomax

#19 por qué?

R

#21 Porque el mantel es infinitamente grande y tiene que quedar completamente teñido de rojo.

japeal

¿Tres? En un punto cualquiera, en un punto de alguno de las circunferencias no coloreadas (es decir a una distancia racional del primer punto). Así solo quedarán sin colorear las intersecciones de los números racionales de los dos primeros grupos de círculos. El tercer punto seria una de estas intersecciones.

fantomax

#27 Primero comentar que está bien darle forma geométrica a la parte sin pintar, circunferencias concéntricas; o he leído demasiado en diagonal o es la primera vez que se menciona.
Pero creo que con tu método se quedan los tres puntos de pinchazo en blanco.

D

#34 Al pinchar la intersección colorea los otros dos... el tercero tengo dudas. Bueno, o uno... pero no los tres.

fantomax

#96 Si está en la intersección es porque dista un racional tanto del primero como del segundo, no colorea ninguno de los 3

D

#27, en tal caso al menos el tercer punto estaría sin colorear. Estaría a distancia racional de los anteriores y de si mismo.

D

Dos veces. Supongamos que un punto está a sqrt(2) de distancia.
Cualquier punto a una distancia x*sqrt(2) con x siendo un número real y no siendo x de la forma sqrt(2)^(2n+1) quedan coloreados. Para colorear estos puntos pincho en uno de ellos y ya quedan automáticamente coloreados puesto entre sí hay una distancia irracional.

1 2